Difference between revisions of "2002 AMC 10P Problems/Problem 22"

(Solution 1)
(Solution 1)
Line 16: Line 16:
  
 
== Solution 1==
 
== Solution 1==
We can solve this problem with an application of Legendre's formula.  
+
We can solve this problem with an application of [[Legendre's Formula|Legendre's Formula]].  
  
 
We know that there will be an abundance of factors of <math>2</math> compared to factors of <math>5,</math> so finding the amount of factors of <math>5</math> is equivalent to finding how many factors of <math>10</math> there are. Additionally, squaring a number will multiply the exponent of each factor by <math>2.</math> Therefore, we plug in <math>p=5</math> and <math>n=2002,</math> then plug in <math>p=5</math> and <math>n=1001</math> and multiply by <math>2</math> in:
 
We know that there will be an abundance of factors of <math>2</math> compared to factors of <math>5,</math> so finding the amount of factors of <math>5</math> is equivalent to finding how many factors of <math>10</math> there are. Additionally, squaring a number will multiply the exponent of each factor by <math>2.</math> Therefore, we plug in <math>p=5</math> and <math>n=2002,</math> then plug in <math>p=5</math> and <math>n=1001</math> and multiply by <math>2</math> in:

Revision as of 07:41, 15 July 2024

Problem

In how many zeroes does the number $\frac{2002!}{(1001!)^2}$ end?

$\text{(A) }0 \qquad \text{(B) }1 \qquad \text{(C) }2 \qquad \text{(D) }200 \qquad \text{(E) }400$

Solution 1

We can solve this problem with an application of Legendre's Formula.

We know that there will be an abundance of factors of $2$ compared to factors of $5,$ so finding the amount of factors of $5$ is equivalent to finding how many factors of $10$ there are. Additionally, squaring a number will multiply the exponent of each factor by $2.$ Therefore, we plug in $p=5$ and $n=2002,$ then plug in $p=5$ and $n=1001$ and multiply by $2$ in:

\[e_p(n!)=\sum_{i=1}^{\infty} \left\lfloor \dfrac{n}{p^i}\right\rfloor =\frac{n-S_{p}(n)}{p-1}\]

\begin{align*} e_5(2002!)=&\left\lfloor\frac{2002}{5}\right\rfloor+\left\lfloor\frac{2002}{5^2}\right\rfloor+\left\lfloor\frac {2002}{5^3}\right\rfloor+\left\lfloor\frac{2002}{5^4}\right\rfloor\\ =&400+80+16+3 \\ =&499 \end{align*}

or alternatively,

$e_5(2002!)=\frac{2002-S_5(2002)}{5-1}=\frac{2002-S_5(31002_5)]}{4}=\frac{2002-6}{4}=499.$

Similarly,

\begin{align*} e_5(2002!)=&\left\lfloor\frac{1001}{5}\right\rfloor+\left\lfloor\frac{1001}{5^2}\right\rfloor+\left\lfloor\frac {1001}{5^3}\right\rfloor+\left\lfloor\frac{1001}{5^4}\right\rfloor\\ =&200+40+8+1 \\ =&299 \end{align*}

or alternatively,

$e_5(1001!)=\frac{1001-S_5(1001)}{5-1}=\frac{1001-S_5(13001_5)]}{4}=\frac{1001-5}{4}=249.$

In any case, our answer is $499-2(249)= \boxed{\textbf{(B) } 1}.$

See also

2002 AMC 10P (ProblemsAnswer KeyResources)
Preceded by
Problem 21
Followed by
Problem 23
1 2 3 4 5 6 7 8 9 10 11 12 13 14 15 16 17 18 19 20 21 22 23 24 25
All AMC 10 Problems and Solutions

The problems on this page are copyrighted by the Mathematical Association of America's American Mathematics Competitions. AMC logo.png